Dadgarnia

New Member
ارسال ها
1,350
لایک ها
1,127
امتیاز
0
#41
پاسخ : ماراتن هندسه

AoPS Forum - MN passes through a constant point- Iran NMO 2005 - Problem2 • Art of Problem Solving
سوال بعد:
روی ضلع BC از متوازی الاضلاع ABCD که A حاده است نقطه T را طوری انتخاب میکنیم که ATD حاده الزاویه شود.اگر مراکز دایره های محیطیه ABT,DAT,CDT را به ترتیب X,Y,Z بنامیم نشان دهید مرکز ارتفاعیه XYZ روی AD قرار دارد.
 

aras2213

New Member
ارسال ها
216
لایک ها
228
امتیاز
0
#42
پاسخ : ماراتن هندسه

3 دایره ی
دو به دو به یکدیگر مماس خارجند.مماس مشترک داخلی
(خط
) دایره ی
را برای بار دوم در P قطع میکند.(یعنی از دو محل تقاطع های
با
، P از

دور تره.)مماس مشترک خارجی
دایره ی
را در
قطع میکند.نشان دهید
نیمساز زاویه ی
است.
 

TheOverlord

New Member
ارسال ها
159
لایک ها
282
امتیاز
0
#43
پاسخ : ماراتن هندسه

انعکاس هم حسابه؟
انعکاس به مرکز محل تماس دوایر
که آن را
مینامیم, دوایر
و خط
را به سه خط موازی تبدیل میکند.حال دو دایره که بر دو خط موازی مماسند یعنی
و خط
یکدیگر را در دونقطه
قطع میکنند. حال کافیست ثابت کنیم
که بدیهی است.

---- دو نوشته به هم متصل شده است ----

فرض کنید ABCD متوازی الاضلاع باشد و نیمساز زاویه A ضلع BC را در E و ضلع CD را در F قطع کند. ثابت کنید که مرکز دایره محیطی CEF روی دایره محیطی CBD قرار دارد.
 

MGH000

New Member
ارسال ها
209
لایک ها
219
امتیاز
0
#44
پاسخ : ماراتن هندسه

دوستان نذارید ماراتنا بخوابه
در مثلث abc داریم:
Ab+bc=3ac
i مرکز دایره محاطی
دایره محاطی در d ,e بر اضلاع ab,ac مماس است.
K,l روبرو قطری های d,e نسبت ب دایره محاطی اند.
ثابت کنید ackl محاطی است.
 

aras2213

New Member
ارسال ها
216
لایک ها
228
امتیاز
0
#45
پاسخ : ماراتن هندسه

دوستان نذارید ماراتنا بخوابه
در مثلث abc داریم:
Ab+bc=3ac
i مرکز دایره محاطی
دایره محاطی در d ,e بر اضلاع ab,ac مماس است.
K,l روبرو قطری های d,e نسبت ب دایره محاطی اند.
ثابت کنید ackl محاطی است.
AoPS Forum - Not hard but cute (points lying on a circle) • Art of Problem Solving

---- دو نوشته به هم متصل شده است ----

انعکاس هم حسابه؟
انعکاس به مرکز محل تماس دوایر
که آن را
مینامیم, دوایر
و خط
را به سه خط موازی تبدیل میکند.حال دو دایره که بر دو خط موازی مماسند یعنی
و خط
یکدیگر را در دونقطه
قطع میکنند. حال کافیست ثابت کنیم
که بدیهی است.

---- دو نوشته به هم متصل شده است ----

فرض کنید ABCD متوازی الاضلاع باشد و نیمساز زاویه A ضلع BC را در E و ضلع CD را در F قطع کند. ثابت کنید که مرکز دایره محیطی CEF روی دایره محیطی CBD قرار دارد.
http://www.artofproblemsolving.com/Forum/viewtopic.php?p=893744&sid=ca1271e8d3ac0b73d36507b 0718e0634#p893744
برعکس اینه که اثباتشون زیاد فرقی نداره

---- دو نوشته به هم متصل شده است ----

سوال بعد:در مثلث ABC ، فرض کنید 'A روبروی قطری A باشد و دایره محاطی داخلی در D,E,F بر BC,AC,AB مماس باشد.اگر A'I پاره خط EF را در H قطع کند نشان دهید DH بر EF عمود است.
 

Dadgarnia

New Member
ارسال ها
1,350
لایک ها
1,127
امتیاز
0
#46
پاسخ : ماراتن هندسه

سوال بعد:در مثلث ABC ، فرض کنید 'A روبروی قطری A باشد و دایره محاطی داخلی در D,E,F بر BC,AC,AB مماس باشد.اگر A'I پاره خط EF را در H قطع کند نشان دهید DH بر EF عمود است.
با راهنمایی های بسیار آقا ارس بالاخره حل شد (کم مونده بود دیگه جوابو بهم بگن :4:) دوستان سعی کنین ماراتن هندسه رو راه بندازیم. [MENTION=14921]aras2213[/MENTION] [MENTION=14457]AHZolfaghari[/MENTION] [MENTION=16947]TheOverlord[/MENTION]
پای عمود
بر
رو
نامگذاری می کنیم و ثابت می کنیم
همخطند. محل برخورد نیمساز های راس های
با دایره ی محیطی رو به ترتیب
و وسط پاره خط
رو
و مرکز دایره ی محیطی رو
می نامیم. می دونیم که مثلث
با مثلث
متجانسه پس مرکز های دایره های محیطی این دو مثلث یعنی
و
با هم متجانسند همچنین می دونیم که
عمود منصف
هه (اثباتش توی کتاب آقای احمد پور هست) پس
مجانس
هست که نتیجه میده
از طرف دیگه طبق عکس قضیه ی تالس توی مثلث
داریم
پس
که همخطی سه نقطه ی
رو نتیجه میده.

---- دو نوشته به هم متصل شده است ----

سوال بعد:
از نقطه ی مفروض K دو مماس KL و KN را بر دایره ی C رسم می کنیم، M را روی امتداد KN پس از N در نظر بگیرید. فرض کنید P نقطه ی تقاطع دایره ی محیطی KLM و دایره ی C و Q پای عمود وارد بر ML از N باشد. ثابت کنید

 

aras2213

New Member
ارسال ها
216
لایک ها
228
امتیاز
0
#47
پاسخ : ماراتن هندسه

فرض کنید ml برای بار دوم c رو در t قطع کنه.در این صورت
.پس


حالا اگه pt ، خط km رو در r قطع کنه،
.همچنین با توجه به قوت نقطه r نسبت به c،

.از این جا به راحتی نتیجه میشود که pqrm محاطی است و





---- دو نوشته به هم متصل شده است ----

سوال بعد:فرض کنید ABC یک مثلث غیر متساوی الساقین باشد که دایره محاطی داخلی آن به ترتیب در D,E,F بر BC,CA,AB مماس است.Q نقطه ای روی دایره محاطی داخلی است به طوری که AQD=90.فرض کنید P نقطه ای درون مثلث باشد به طوری که روی AI باشد و MD=MP.(نقطه M وسط ضلع BC است.)
نشان دهید یکی از زاویه های PQE یا PQF برابر 90 است.

**M رو نگفته بودم چیه:4:،اضافه شد.
 
آخرین ویرایش توسط مدیر

TheOverlord

New Member
ارسال ها
159
لایک ها
282
امتیاز
0
#48
پاسخ : ماراتن هندسه

لم: Q روي دايره به مركز m و شعاع md است.
اثبات: محل برخورد aq با دايره محاطي را d' بگيريد . پس d'qd=90 يعني d' روبرو قطري d در دايره محاطي است. اما لم معروفي بيان ميكند كه محل برخورد d'a و bc پاي تماس دايره محاطي خارجي است پس اگر k باشد آنگاه mk=md ولي kqd عمود است پس مثلث kqd قائم الزاويه است پس مركز دايره محيطي آن وسط وتر يعني m است
لم دو: اگر نام محل برخورد ميان خط نظير ac و ai را l بگيريم، l هم روي همان دايره مذكور است.
اثبات : طول ml مستقيما به دست مي آيد.
به طريق مشابه t محل برخورد ميان خط نظير ab و ai هم روي همان دايره مذكور است. از طرفي چون دايره مذكور با خط ai حد اكثر در دو نقطه تقاطع ميكند البته طبق تعريف p نيز محل برخورد ai و آن دايره است پس p=l يا p=t. بنابر تقارن فرض كنيد p=l. ثابت ميكنيم pqe=90. چون pqe=lqe=eqd-lqd=180-efd-lmd/2=180-(90-c/2)-c/2=90

كه حكم مساله است.

سوال بعد: در مثلث i ،abc مرکز دايره محاطی داخلی است. عمودی که بر ai از نقطه i خارج میشود اضلاع ac و ab را به ترتيب در ′b و ′c قطع میکند. نقطه های b۱ و c۱ به ترتيب روی نيم خط های bc و cb طوری هستند که ab = bb۱و ac = cc۱. اگر t نقطه دوم برخورد دايره های محيطی مثلث های ′ab۱c و ′ac۱b باشد، ثابت کنيد مرکز دايره محيطی مثلث ati روي bc است.
 

aras2213

New Member
ارسال ها
216
لایک ها
228
امتیاز
0
#49
پاسخ : ماراتن هندسه

لم: Q روي دايره به مركز m و شعاع md است.
اثبات: محل برخورد aq با دايره محاطي را d' بگيريد . پس d'qd=90 يعني d' روبرو قطري d در دايره محاطي است. اما لم معروفي بيان ميكند كه محل برخورد d'a و bc پاي تماس دايره محاطي خارجي است پس اگر k باشد آنگاه mk=md ولي kqd عمود است پس مثلث kqd قائم الزاويه است پس مركز دايره محيطي آن وسط وتر يعني m است
لم دو: اگر نام محل برخورد ميان خط نظير ac و ai را l بگيريم، l هم روي همان دايره مذكور است.
اثبات : طول ml مستقيما به دست مي آيد.
به طريق مشابه t محل برخورد ميان خط نظير ab و ai هم روي همان دايره مذكور است. از طرفي چون دايره مذكور با خط ai حد اكثر در دو نقطه تقاطع ميكند البته طبق تعريف p نيز محل برخورد ai و آن دايره است پس p=l يا p=t. بنابر تقارن فرض كنيد p=l. ثابت ميكنيم pqe=90. چون pqe=lqe=eqd-lqd=180-efd-lmd/2=180-(90-c/2)-c/2=90

كه حكم مساله است.

سوال بعد: در مثلث i ،abc مرکز دايره محاطی داخلی است. عمودی که بر ai از نقطه i خارج میشود اضلاع ac و ab را به ترتيب در ′b و ′c قطع میکند. نقطه های b۱ و c۱ به ترتيب روی نيم خط های bc و cb طوری هستند که ab = bb۱و ac = cc۱. اگر t نقطه دوم برخورد دايره های محيطی مثلث های ′ab۱c و ′ac۱b باشد، ثابت کنيد مرکز دايره محيطی مثلث ati روي bc است.
AoPS Forum - question6 • Art of Problem Solving
 

aras2213

New Member
ارسال ها
216
لایک ها
228
امتیاز
0
#51
پاسخ : ماراتن هندسه

فكر كردم اوني كه تو مثلينكس نيست رو گذاشتم.
:4::4:

سوال بعد:فرض کنید یک نقطه تقاطع دایره های
، نقطه
باشد.یکی از مماس مشترک های این دو دایره
را به ترتیب در
قطع میکند.فرض کنید عمود وارد از
به
،
خط
را در
قطع کند.نشان دهید
.
 

TheOverlord

New Member
ارسال ها
159
لایک ها
282
امتیاز
0
#52
پاسخ : ماراتن هندسه

فروض مساله را به صورت ضرب برداری مینویسیم.فرض کنید
مبدا مختصات است.

که در آن k عددی حقیقی است و بدیهتا یک نیست. حکم نیز به این صورت میشود:

اما داشتیم
پس کافیست بگوییم

حال رابطه هایی که از برابری طولی شعاع ها به دست آمد را باز میکنیم:

حال این دو رابطه را تفریق میکنیم:

از باز کردن رابطه های نظیر عمود بودن شعاع بر مماس نیز داریم:

حال دو رابطه ای که الان باز کردیم را جمع میکنیم:

پس

پس

که ثابت کردیم معادل حکم مساله است.

---- دو نوشته به هم متصل شده است ----

سوال بعد:دو دایره
مماس خارج در نقطه
هستند.
نقطه ای روی
است که روی خط المرکزین دوایر نیست. فرض کنید
بر
مماس باشند.
دایره
را در نقاط
قطع میکند. ثابت کنید خط
, مماس بر
در نقطه
و مماس مشترک داخلی دو دایره همرسند.
 

aras2213

New Member
ارسال ها
216
لایک ها
228
امتیاز
0
#53
پاسخ : ماراتن هندسه

میخوایم نشون بدیم AP,FE)=-1).پس باید بگیم که AF/AE=PF/PE.حالا اگه AP رو امتداد بدیم تا w_2 رو در Q قطع کنه، 1-=(PQ,BC).از طرفی با زاویه بازی میتونیم نشون بدیم که AFP ,BQC متشابهن.همچنین FPE,BPC هم متشابهن.پس نتیجه میشه که AF/AE=QC/QB=PC/PB=PF/PE که حکم رو نتیجه میده.

---- دو نوشته به هم متصل شده است ----

پویا ببخشید من نمیفهمم ولی توی این
وقتی علامت بردار روی p میاد یعنی چی؟:96:
 
آخرین ویرایش توسط مدیر

TheOverlord

New Member
ارسال ها
159
لایک ها
282
امتیاز
0
#54
پاسخ : ماراتن هندسه

بردار يه نقطه، يعني برداري كه از مبدا به اون نقطه رسم ميكنيم گرچه مطمئنم منظورت اين نيست.
 

TheOverlord

New Member
ارسال ها
159
لایک ها
282
امتیاز
0
#56
پاسخ : ماراتن هندسه

سوال بعد: در چهارضلعي غير ذوزنقه abcd, محل برخورد امتداد ab و cd را e و محل برخورد امتداد ad و bc را f گذاشته ايم.
P نقطه اي داخل چهارضلعي است بطوري كهfpb=epd. ثابت كنيد epc+fpa=180
 

TheOverlord

New Member
ارسال ها
159
لایک ها
282
امتیاز
0
#57
پاسخ : ماراتن هندسه

رو امتداد ميديم تا ضلع
رو در
قطع كنه. كافيه بگيم
اما معادل اينه كه A,Q نسبت به نيمساز زاويه
مزدوج همزاويه باشند.اما اين معادل اينه كه P(EADB)=P(FQBD) چون مزدوج همزاويه كردن ناهمساز رو حفط ميكنه.فرض كنيد PB خطوط CD و AD رو در R و S قطع كنه. همچنين PE خط AD رو در T قطع ميكنه. پس P(EDAB)=(TDAS)=(PRBS)=(QDFS)=P(QDFB)=P(FQBD) كه همون حكم مساله است.

---- دو نوشته به هم متصل شده است ----

سوال بعد: در مثلث ABC داريم AB=AC .نقطه D وسط BC است. E پاي عمود D بر AC است.F وسط DE است. ثابت كنيد AF بر BE عمود است.

ويرايش: راس متساوي الساقين رو اشتباها B گذاشته بودم.
 
آخرین ویرایش توسط مدیر

aras2213

New Member
ارسال ها
216
لایک ها
228
امتیاز
0
#58
پاسخ : ماراتن هندسه

سوال بعد:مثلث متساوی الساقین (ab=ac) مفروض است و k روی ضلع bc . قرینه ی k رو نسبت به b , c به ترتیب p , q می نامیم. دایره ی w را از k می گذرانیم که بر ضلع های ab , ac مماس باشد. این دایره ak را در m قطع می کند. ثابت کنید دایره ی محیطی pqm بر w مماس است.(من حل قبلی رو نخوندم ولی به نظر درست میاد.)
 
آخرین ویرایش توسط مدیر

Dadgarnia

New Member
ارسال ها
1,350
لایک ها
1,127
امتیاز
0
#59
پاسخ : ماراتن هندسه

سوال بعد:مثلث متساوی الساقین (ab=ac) مفروض است و k روی ضلع bc . قرینه ی k رو نسبت به b , c به ترتیب p , q می نامیم. دایره ی w را از k می گذرانیم که بر ضلع های ab , ac مماس باشد. این دایره ak را در m قطع می کند. ثابت کنید دایره ی محیطی pqm بر w مماس است.(من حل قبلی رو نخوندم ولی به نظر درست میاد.)
ببخشید ولی مثل اینکه آقای پویا سوال رو اشتباه گذاشته بودن و الان درستش کردن و سوال قبل همچنان حل نشده است!
 

TheOverlord

New Member
ارسال ها
159
لایک ها
282
امتیاز
0
#60
پاسخ : ماراتن هندسه

ببخشید ولی مثل اینکه آقای پویا سوال رو اشتباه گذاشته بودن و الان درستش کردن و سوال قبل همچنان حل نشده است!
و البته اون سوال غلط بود. نميدونم چرا ايشون حل كردن؟
 
بالا